Is expected value $E[X_1;X_1leq X_2]= E[X_1;X_1< X_2]$Find $E(X_1|X_2leq x_2, X_3leq x_3)$ where $(X_1,X_2,X_3)$ is multivariate normalConditional expectation: $E[X_1 X_2mid X_1 + X_2 X_3]$Probability and expectation of three ordered random variablesFind the value of $mathbbE(X_1+X_2+ldots+X_N)$ of i.i.d random variables $X_i$s.Verifying calculation inside an expected value problemLet $X_1,X_2,X_3$ be iid. U($0,1$) random variables. Then what will be the value of $E(fracX_1+X_2X_1+X_2+X_3$)?Throw a dice-expected value.Finding conditional expectation $E[X_1 | X_2 = x_2]$Expected value- $E(1/3)^t$Expected value of $Z=X_1+X_2$ if $X_1<X_3$.and $Z=X_1$ if $X_3leq X_1$

Is it true that real estate prices mainly go up?

Is "history" a male-biased word ("his+story")?

What is the meaning of triple curly braces in phtml template files? When and how do we use them?

Solving "Resistance between two nodes on a grid" problem in Mathematica

Why don't MCU characters ever seem to have language issues?

Good for you! in Russian

A question on the ultrafilter number

Does a Catoblepas statblock appear in an official D&D 5e product?

"One can do his homework in the library"

MTG: Can I kill an opponent in response to lethal activated abilities, and not take the damage?

Should QA ask requirements to developers?

Algorithm to convert a fixed-length string to the smallest possible collision-free representation?

Why is there a voltage between the mains ground and my radiator?

PTIJ: Why can't I eat anything?

How much stiffer are 23c tires over 28c?

Am I not good enough for you?

They call me Inspector Morse

If the Captain's screens are out, does he switch seats with the co-pilot?

Why does Deadpool say "You're welcome, Canada," after shooting Ryan Reynolds in the end credits?

Do I really need to have a scientific explanation for my premise?

Finding algorithms of QGIS commands?

Do f-stop and exposure time perfectly cancel?

What to do when during a meeting client people start to fight (even physically) with each others?

What wound would be of little consequence to a biped but terrible for a quadruped?



Is expected value $E[X_1;X_1leq X_2]= E[X_1;X_1


Find $E(X_1|X_2leq x_2, X_3leq x_3)$ where $(X_1,X_2,X_3)$ is multivariate normalConditional expectation: $E[X_1 X_2mid X_1 + X_2 X_3]$Probability and expectation of three ordered random variablesFind the value of $mathbbE(X_1+X_2+ldots+X_N)$ of i.i.d random variables $X_i$s.Verifying calculation inside an expected value problemLet $X_1,X_2,X_3$ be iid. U($0,1$) random variables. Then what will be the value of $E(fracX_1+X_2X_1+X_2+X_3$)?Throw a dice-expected value.Finding conditional expectation $E[X_1 | X_2 = x_2]$Expected value- $E(1/3)^t$Expected value of $Z=X_1+X_2$ if $X_1<X_3$.and $Z=X_1$ if $X_3leq X_1$













0












$begingroup$


Let $X_1, X_2$ two independent random variables with PDF $f_X_i(x_i)$.
Is this formula is true
$$E[X_1;X_1leq X_2]=
int_x_2=0^inftyBig(int_x_1=0^x_2x_1 f_X_1(x_1)d x_1Big)f_X_2(x_2)dx_2$$



I am asking if the expected value
$$E[X_1;X_1leq X_2]= E[X_1;X_1< X_2]$$,



or
$$E[X_1;X_1< X_2]=E[X_1]-E[X_1;X_1leq X_2].$$



Thanks










share|cite|improve this question









$endgroup$
















    0












    $begingroup$


    Let $X_1, X_2$ two independent random variables with PDF $f_X_i(x_i)$.
    Is this formula is true
    $$E[X_1;X_1leq X_2]=
    int_x_2=0^inftyBig(int_x_1=0^x_2x_1 f_X_1(x_1)d x_1Big)f_X_2(x_2)dx_2$$



    I am asking if the expected value
    $$E[X_1;X_1leq X_2]= E[X_1;X_1< X_2]$$,



    or
    $$E[X_1;X_1< X_2]=E[X_1]-E[X_1;X_1leq X_2].$$



    Thanks










    share|cite|improve this question









    $endgroup$














      0












      0








      0





      $begingroup$


      Let $X_1, X_2$ two independent random variables with PDF $f_X_i(x_i)$.
      Is this formula is true
      $$E[X_1;X_1leq X_2]=
      int_x_2=0^inftyBig(int_x_1=0^x_2x_1 f_X_1(x_1)d x_1Big)f_X_2(x_2)dx_2$$



      I am asking if the expected value
      $$E[X_1;X_1leq X_2]= E[X_1;X_1< X_2]$$,



      or
      $$E[X_1;X_1< X_2]=E[X_1]-E[X_1;X_1leq X_2].$$



      Thanks










      share|cite|improve this question









      $endgroup$




      Let $X_1, X_2$ two independent random variables with PDF $f_X_i(x_i)$.
      Is this formula is true
      $$E[X_1;X_1leq X_2]=
      int_x_2=0^inftyBig(int_x_1=0^x_2x_1 f_X_1(x_1)d x_1Big)f_X_2(x_2)dx_2$$



      I am asking if the expected value
      $$E[X_1;X_1leq X_2]= E[X_1;X_1< X_2]$$,



      or
      $$E[X_1;X_1< X_2]=E[X_1]-E[X_1;X_1leq X_2].$$



      Thanks







      conditional-expectation expected-value






      share|cite|improve this question













      share|cite|improve this question











      share|cite|improve this question




      share|cite|improve this question










      asked 2 days ago









      MonirMonir

      368




      368




















          1 Answer
          1






          active

          oldest

          votes


















          1












          $begingroup$

          If $X_1$ and $X_2$ are independent random variables with a densities then $PX_1=X_2=0$ so it is true that $E(X_1;X_1 leq X_2)=E(X_1;X_1 < X_2)$.






          share|cite|improve this answer









          $endgroup$












          • $begingroup$
            Ok, is my formula is true $$E[X_1;X_1leq X_2]= int_x_2=0^inftyBig(int_x_1=0^x_2x_1 f_X_1(x_1)d x_1Big)f_X_2(x_2)dx_2$$
            $endgroup$
            – Monir
            2 days ago










          • $begingroup$
            @Monir It is true provided the random variables are non-negative. In general the integrals start from $-infty$.
            $endgroup$
            – Kavi Rama Murthy
            2 days ago










          • $begingroup$
            Ok, thank you Prof Kavi Rama Murthy.
            $endgroup$
            – Monir
            2 days ago










          Your Answer





          StackExchange.ifUsing("editor", function ()
          return StackExchange.using("mathjaxEditing", function ()
          StackExchange.MarkdownEditor.creationCallbacks.add(function (editor, postfix)
          StackExchange.mathjaxEditing.prepareWmdForMathJax(editor, postfix, [["$", "$"], ["\\(","\\)"]]);
          );
          );
          , "mathjax-editing");

          StackExchange.ready(function()
          var channelOptions =
          tags: "".split(" "),
          id: "69"
          ;
          initTagRenderer("".split(" "), "".split(" "), channelOptions);

          StackExchange.using("externalEditor", function()
          // Have to fire editor after snippets, if snippets enabled
          if (StackExchange.settings.snippets.snippetsEnabled)
          StackExchange.using("snippets", function()
          createEditor();
          );

          else
          createEditor();

          );

          function createEditor()
          StackExchange.prepareEditor(
          heartbeatType: 'answer',
          autoActivateHeartbeat: false,
          convertImagesToLinks: true,
          noModals: true,
          showLowRepImageUploadWarning: true,
          reputationToPostImages: 10,
          bindNavPrevention: true,
          postfix: "",
          imageUploader:
          brandingHtml: "Powered by u003ca class="icon-imgur-white" href="https://imgur.com/"u003eu003c/au003e",
          contentPolicyHtml: "User contributions licensed under u003ca href="https://creativecommons.org/licenses/by-sa/3.0/"u003ecc by-sa 3.0 with attribution requiredu003c/au003e u003ca href="https://stackoverflow.com/legal/content-policy"u003e(content policy)u003c/au003e",
          allowUrls: true
          ,
          noCode: true, onDemand: true,
          discardSelector: ".discard-answer"
          ,immediatelyShowMarkdownHelp:true
          );



          );













          draft saved

          draft discarded


















          StackExchange.ready(
          function ()
          StackExchange.openid.initPostLogin('.new-post-login', 'https%3a%2f%2fmath.stackexchange.com%2fquestions%2f3142277%2fis-expected-value-ex-1x-1-leq-x-2-ex-1x-1-x-2%23new-answer', 'question_page');

          );

          Post as a guest















          Required, but never shown

























          1 Answer
          1






          active

          oldest

          votes








          1 Answer
          1






          active

          oldest

          votes









          active

          oldest

          votes






          active

          oldest

          votes









          1












          $begingroup$

          If $X_1$ and $X_2$ are independent random variables with a densities then $PX_1=X_2=0$ so it is true that $E(X_1;X_1 leq X_2)=E(X_1;X_1 < X_2)$.






          share|cite|improve this answer









          $endgroup$












          • $begingroup$
            Ok, is my formula is true $$E[X_1;X_1leq X_2]= int_x_2=0^inftyBig(int_x_1=0^x_2x_1 f_X_1(x_1)d x_1Big)f_X_2(x_2)dx_2$$
            $endgroup$
            – Monir
            2 days ago










          • $begingroup$
            @Monir It is true provided the random variables are non-negative. In general the integrals start from $-infty$.
            $endgroup$
            – Kavi Rama Murthy
            2 days ago










          • $begingroup$
            Ok, thank you Prof Kavi Rama Murthy.
            $endgroup$
            – Monir
            2 days ago















          1












          $begingroup$

          If $X_1$ and $X_2$ are independent random variables with a densities then $PX_1=X_2=0$ so it is true that $E(X_1;X_1 leq X_2)=E(X_1;X_1 < X_2)$.






          share|cite|improve this answer









          $endgroup$












          • $begingroup$
            Ok, is my formula is true $$E[X_1;X_1leq X_2]= int_x_2=0^inftyBig(int_x_1=0^x_2x_1 f_X_1(x_1)d x_1Big)f_X_2(x_2)dx_2$$
            $endgroup$
            – Monir
            2 days ago










          • $begingroup$
            @Monir It is true provided the random variables are non-negative. In general the integrals start from $-infty$.
            $endgroup$
            – Kavi Rama Murthy
            2 days ago










          • $begingroup$
            Ok, thank you Prof Kavi Rama Murthy.
            $endgroup$
            – Monir
            2 days ago













          1












          1








          1





          $begingroup$

          If $X_1$ and $X_2$ are independent random variables with a densities then $PX_1=X_2=0$ so it is true that $E(X_1;X_1 leq X_2)=E(X_1;X_1 < X_2)$.






          share|cite|improve this answer









          $endgroup$



          If $X_1$ and $X_2$ are independent random variables with a densities then $PX_1=X_2=0$ so it is true that $E(X_1;X_1 leq X_2)=E(X_1;X_1 < X_2)$.







          share|cite|improve this answer












          share|cite|improve this answer



          share|cite|improve this answer










          answered 2 days ago









          Kavi Rama MurthyKavi Rama Murthy

          66.6k53067




          66.6k53067











          • $begingroup$
            Ok, is my formula is true $$E[X_1;X_1leq X_2]= int_x_2=0^inftyBig(int_x_1=0^x_2x_1 f_X_1(x_1)d x_1Big)f_X_2(x_2)dx_2$$
            $endgroup$
            – Monir
            2 days ago










          • $begingroup$
            @Monir It is true provided the random variables are non-negative. In general the integrals start from $-infty$.
            $endgroup$
            – Kavi Rama Murthy
            2 days ago










          • $begingroup$
            Ok, thank you Prof Kavi Rama Murthy.
            $endgroup$
            – Monir
            2 days ago
















          • $begingroup$
            Ok, is my formula is true $$E[X_1;X_1leq X_2]= int_x_2=0^inftyBig(int_x_1=0^x_2x_1 f_X_1(x_1)d x_1Big)f_X_2(x_2)dx_2$$
            $endgroup$
            – Monir
            2 days ago










          • $begingroup$
            @Monir It is true provided the random variables are non-negative. In general the integrals start from $-infty$.
            $endgroup$
            – Kavi Rama Murthy
            2 days ago










          • $begingroup$
            Ok, thank you Prof Kavi Rama Murthy.
            $endgroup$
            – Monir
            2 days ago















          $begingroup$
          Ok, is my formula is true $$E[X_1;X_1leq X_2]= int_x_2=0^inftyBig(int_x_1=0^x_2x_1 f_X_1(x_1)d x_1Big)f_X_2(x_2)dx_2$$
          $endgroup$
          – Monir
          2 days ago




          $begingroup$
          Ok, is my formula is true $$E[X_1;X_1leq X_2]= int_x_2=0^inftyBig(int_x_1=0^x_2x_1 f_X_1(x_1)d x_1Big)f_X_2(x_2)dx_2$$
          $endgroup$
          – Monir
          2 days ago












          $begingroup$
          @Monir It is true provided the random variables are non-negative. In general the integrals start from $-infty$.
          $endgroup$
          – Kavi Rama Murthy
          2 days ago




          $begingroup$
          @Monir It is true provided the random variables are non-negative. In general the integrals start from $-infty$.
          $endgroup$
          – Kavi Rama Murthy
          2 days ago












          $begingroup$
          Ok, thank you Prof Kavi Rama Murthy.
          $endgroup$
          – Monir
          2 days ago




          $begingroup$
          Ok, thank you Prof Kavi Rama Murthy.
          $endgroup$
          – Monir
          2 days ago

















          draft saved

          draft discarded
















































          Thanks for contributing an answer to Mathematics Stack Exchange!


          • Please be sure to answer the question. Provide details and share your research!

          But avoid


          • Asking for help, clarification, or responding to other answers.

          • Making statements based on opinion; back them up with references or personal experience.

          Use MathJax to format equations. MathJax reference.


          To learn more, see our tips on writing great answers.




          draft saved


          draft discarded














          StackExchange.ready(
          function ()
          StackExchange.openid.initPostLogin('.new-post-login', 'https%3a%2f%2fmath.stackexchange.com%2fquestions%2f3142277%2fis-expected-value-ex-1x-1-leq-x-2-ex-1x-1-x-2%23new-answer', 'question_page');

          );

          Post as a guest















          Required, but never shown





















































          Required, but never shown














          Required, but never shown












          Required, but never shown







          Required, but never shown

































          Required, but never shown














          Required, but never shown












          Required, but never shown







          Required, but never shown







          Popular posts from this blog

          Lowndes Grove History Architecture References Navigation menu32°48′6″N 79°57′58″W / 32.80167°N 79.96611°W / 32.80167; -79.9661132°48′6″N 79°57′58″W / 32.80167°N 79.96611°W / 32.80167; -79.9661178002500"National Register Information System"Historic houses of South Carolina"Lowndes Grove""+32° 48' 6.00", −79° 57' 58.00""Lowndes Grove, Charleston County (260 St. Margaret St., Charleston)""Lowndes Grove"The Charleston ExpositionIt Happened in South Carolina"Lowndes Grove (House), Saint Margaret Street & Sixth Avenue, Charleston, Charleston County, SC(Photographs)"Plantations of the Carolina Low Countrye

          random experiment with two different functions on unit interval Announcing the arrival of Valued Associate #679: Cesar Manara Planned maintenance scheduled April 23, 2019 at 00:00UTC (8:00pm US/Eastern)Random variable and probability space notionsRandom Walk with EdgesFinding functions where the increase over a random interval is Poisson distributedNumber of days until dayCan an observed event in fact be of zero probability?Unit random processmodels of coins and uniform distributionHow to get the number of successes given $n$ trials , probability $P$ and a random variable $X$Absorbing Markov chain in a computer. Is “almost every” turned into always convergence in computer executions?Stopped random walk is not uniformly integrable

          How should I support this large drywall patch? Planned maintenance scheduled April 23, 2019 at 00:00UTC (8:00pm US/Eastern) Announcing the arrival of Valued Associate #679: Cesar Manara Unicorn Meta Zoo #1: Why another podcast?How do I cover large gaps in drywall?How do I keep drywall around a patch from crumbling?Can I glue a second layer of drywall?How to patch long strip on drywall?Large drywall patch: how to avoid bulging seams?Drywall Mesh Patch vs. Bulge? To remove or not to remove?How to fix this drywall job?Prep drywall before backsplashWhat's the best way to fix this horrible drywall patch job?Drywall patching using 3M Patch Plus Primer